pT 30 # 22 Forum

Prepare for the LSAT or discuss it with others in this forum.
Post Reply
roballen

New
Posts: 55
Joined: Wed Jun 23, 2010 9:34 am

pT 30 # 22

Post by roballen » Wed Aug 18, 2010 2:38 pm

Can someone please explain why this is D?

Is it right bc the ppl who had their normal daily caloric intake increased by 25% ( lets say 2000 calorie diet) they would then consume 2,500 calories while the other group still consumed the 2,000 calories but 25% of that consumption now was alcohol and each gained the same amt of weight even though one group was having 500 more calories.

User avatar
Anaconda

Silver
Posts: 605
Joined: Sun Jun 06, 2010 3:51 pm

Re: pT 30 # 22

Post by Anaconda » Wed Aug 18, 2010 2:54 pm

roballen wrote:Can someone please explain why this is D?

Is it right bc the ppl who had their normal daily caloric intake increased by 25% ( lets say 2000 calorie diet) they would then consume 2,500 calories while the other group still consumed the 2,000 calories but 25% of that consumption now was alcohol and each gained the same amt of weight even though one group was having 500 more calories.
You got it.

A Is incorrect because it's not compared to any other food or drinks, although it's a tricky answer choice.

B contradicts the results of the study.

C is wrong - body weight is never mentioned in the stimulus and is out of scope.

D is correct - if one consumes 25% more calories yet doesn't gain any more body fat, than maybe
other factors contribute to body fat.

E is not at all proven by the stimulus - they both put on the same amount of body fat despite consuming different proportions of alcohol in the caloric intake. Also, what about diets is mentioned in the stimulus?

This is a tough problem for sure, but if you efficiently scrutinize the answer choices, it shouldn't take you too long to narrow it all the way down to D. Remember inference questions can't introduce new info, so that eliminates C, and B and E are opposite answer choices, so you already have it narrowed down to A and D.

Also, it might be easier to mentally reword "most strongly supported" questions as "we can conclude from the stimulus that..." - most strongly supported implies there could be a few credible answer choices, but in reality most strongly supported questions are the same as must be true/inference questions - the answer is the only answer choice PROVEN by the stimulus, the rest are all not supported or contradict the stimulus, as shown in this question.

Post Reply

Return to “LSAT Prep and Discussion Forum”